LSAT and Law School Admissions Forum

Get expert LSAT preparation and law school admissions advice from PowerScore Test Preparation.

 Administrator
PowerScore Staff
  • PowerScore Staff
  • Posts: 8916
  • Joined: Feb 02, 2011
|
#34931
Complete Question Explanation

Point at Issue. The correct answer choice is (C)

In this dialogue the two speakers discuss what should be done if, as the mayor says, the city can
no longer fund both children’s educational projects: the children’s museum and local children’s
television programming. Alissa asserts that if the city cannot afford to fund both projects, it should
opt to continue funding the museum, because the children’s museum exhibits are much more
interactive than passive television-viewing, providing richer experience as a result.
Greta disagrees with the assertion that the city should continue to fund the children’s television
programming rather than the museum, because the kids’ television shows reach a much wider
audience.

The two speakers’ arguments break down simply as follows:

Alissa:
..... Premise: ..... The interactive exhibits at the children’s museum create richer educational
..... ..... ..... ..... experience for the children.

..... Conclusion: ..... If the city cannot afford to continue funding both projects, it should choose the
..... ..... ..... ..... museum, and cease funding the television programs.

Greta:

..... Premise: ..... The museum reaches a much smaller audience.

..... Conclusion: ..... Thus, if the city cannot afford both projects, it should choose to continue to
..... ..... ..... ..... fund the children’s television programming, and cease funding the museum.

The stimulus is followed by a Point at Issue question, so the correct answer choice will provide a
statement with which one speaker will clearly agree and the other will clearly disagree. Specifically,
the correct answer choice in this case will relay the disagreement between Alissa and Greta:
Between the two projects, Alissa thinks that the museum is the one that deserves continued funding
from the city, and Greta is of the opinion that the city should continue its local children’s television
programming, and stop funding the children’s museum.

Answer choice (A): Each of the speakers discusses what must happen if the city cannot afford to
continue funding both projects. Since neither speaker refers to the issue presented in this answer
choice, it cannot be the correct answer to this Point at Issue question.

Answer choice (B): Neither speaker questions the truthfulness of the mayor’s words; they simply
disagree about which project should no longer receive funding from the city.

Answer choice (C): This is the correct answer choice. As discussed, the two speakers disagree
over the issue of which project should enjoy funding in the case that the city cannot afford to
continue funding both.

Answer choice (D): There is no debate about the wider audience among television viewers; Greta is
the only one who mentions this, and Alissa does not take issue. The debate comes down to whether
a more interactive experience or a wider audience is more important, and which funding decision the
city should make.

Answer choice (E): Alissa makes the point that the museum’s greater degree of interactivity makes
for a richer educational experience for the children, but Greta does not refute this. Rather, Greta
brings up the point that television reaches a bigger local audience, arguing that the museum is the
project that should have its funding discontinued. Since this is not the point at issue between the
speakers, this choice should be ruled out of contention.
 norvo
  • Posts: 2
  • Joined: Nov 11, 2014
|
#17514
Hi Guys!

I was torn between A and C, but picked A. Why is C correct?

Thank you.
 Ron Gore
PowerScore Staff
  • PowerScore Staff
  • Posts: 220
  • Joined: May 15, 2013
|
#17537
Hi Norvo,

In this Point at Issue question, the only thing that both speakers talk about is which program should no longer be funded if it is the case that the city can no longer afford to fund both.

Answer choice (A) is incorrect because Alissa does not take a position on whether or not the city can, in fact, fund both programs.

Answer choice (C) is correct because it describes what the two speakers disagree about: which if the programs should be defunded if the city cannot afford both.

Thanks,

Ron
User avatar
 lemonade42
  • Posts: 42
  • Joined: Feb 23, 2024
|
#106037
Hi in response to the previous reply,

Isn't (A )wrong because Greta's position is unknown about what happens if the children's museum is continued to be funded. Because I would think that Alissa would agree with (A) and say that the TV programming funding should be ceased because she said so directly in the stimulus.
 Luke Haqq
PowerScore Staff
  • PowerScore Staff
  • Posts: 747
  • Joined: Apr 26, 2012
|
#106047
Hi lemonade42!

What Alissa does in this stimulus is reports what the mayor says: "If, as the mayor says, the city can no longer continue to fund both the children's museum and local children's television programming, then it should cease funding the television programming." Notice that she doesn't affirm that she believes the mayor is correct. Rather, she only says "if" what the mayor says is true, then XYZ should happen.

We therefore don't know what Alissa's position would be with respect to answer choice (A). The same is true of Greta, as you point out. We don't know Greta's position on answer choice (A). Since we don't know either of their positions, it can't be a point of disagreement between them.

Get the most out of your LSAT Prep Plus subscription.

Analyze and track your performance with our Testing and Analytics Package.